0% found this document useful (0 votes)
33 views33 pages

File 825

1) The mass of the car was converted to 1400 kg from newtons and then to 95.9 slugs. 2) Using the equation for weight (W=mg), the weight of the car in newtons was calculated to be 13,730 N. 3) The weight of the car in pounds was then calculated from its mass in slugs (95.9 slugs) to be 3,090 lb.

Uploaded by

Amin Encek
Copyright
© © All Rights Reserved
We take content rights seriously. If you suspect this is your content, claim it here.
Available Formats
Download as PDF, TXT or read online on Scribd
0% found this document useful (0 votes)
33 views33 pages

File 825

1) The mass of the car was converted to 1400 kg from newtons and then to 95.9 slugs. 2) Using the equation for weight (W=mg), the weight of the car in newtons was calculated to be 13,730 N. 3) The weight of the car in pounds was then calculated from its mass in slugs (95.9 slugs) to be 3,090 lb.

Uploaded by

Amin Encek
Copyright
© © All Rights Reserved
We take content rights seriously. If you suspect this is your content, claim it here.
Available Formats
Download as PDF, TXT or read online on Scribd
You are on page 1/ 33

‫ﺑﺴﻤﻪ ﺗﻌﺎﻟﯽ‬

‫ﻣﻮاد آﻣﻮزﺷﯽ درس اﺳﺘﺎﺗﯿﮏ‬

‫‪98/12/17‬‬ ‫اﺳﺘﺎد درس‪ :‬ﺑﺎﻏﺴﺘﺎﻧﯽ‬

‫داﻧﺸﺠﻮﯾﺎن ﻣﺤﺘﺮم درس اﺳﺘﺎﺗﯿﮏ‪:‬‬


‫ﺑﺎ ﺳﻼم‪ .‬ﺑﺎ ﺗﻮﺟﻪ ﺑﻪ اﻫﻤﯿﺖ درس اﺳﺘﺎﺗﯿﮏ و ﺷﺮاﯾﻂ ﭘﯿﺶ آﻣﺪه‪ ،‬ﻻزم اﺳﺖ داﻧﺸﺠﻮﯾﺎن ﻣﺤﺘﺮم ﻋﻼوه ﺑﺮ اﺳـﺘﻔﺎده از‬
‫وﯾﺪﺋﻮﻫﺎي ﺗﻬﯿﻪ ﺷﺪه‪ ،‬ﺧﻮد ﻧﯿﺰ ﺑﻪ ﻣﻄﺎﻟﻌـﻪ درس از روي ﮐﺘـﺎب ﺑﭙﺮدازﻧـﺪ‪ .‬ﻫـﻢﭼﻨـﯿﻦ ﺣـﻞ ﻣﺴـﺎﯾﻞ ﻣﺨﺘﻠـﻒ ﺑـﺮاي‬
‫ﯾﺎدﮔﯿﺮي درس اﺳﺘﺎﺗﯿﮏ ﺿﺮوري اﺳﺖ‪ .‬ﻟﺬا ﺑﺮاي ﺳﺎده ﺗﺮ ﺷﺪن دﺳﺘﺮﺳﯽ ﺑﻪ ﺳﻮاﻻت ﻣﺨﺘﻠﻒ‪ ،‬ﻣﺜﺎلﻫﺎي ﺣـﻞ ﺷـﺪه‬
‫ﻓﺼﻮل ﯾﮏ ﺗﺎ ﺳﻪ ﮐﺘﺎب اﺳﺘﺎﺗﯿﮏ ﻣﺮﯾﺎم در اﯾﻨﺠﺎ ﺟﻤﻊآوري ﺷﺪه اﺳﺖ‪ .‬اﺑﺘﺪا ﺗﺮﺟﻤﻪ ﻓﺎرﺳﯽ ﺳﻮاﻻت و ﺳﭙﺲ ﺳـﻮال‬
‫و ﺣﻞ آن از روي ﮐﺘﺎب آورده ﺷﺪه اﺳﺖ‪ .‬ﭘﺲ از ﻣﻄﺎﻟﻌﻪ و ﯾﺎدﮔﯿﺮي ﻫﺮ ﺑﺨﺶ اﺑﺘﺪا ﺳﻌﯽ ﮐﻨﯿـﺪ ﺧﻮدﺗـﺎن ﺳـﻮال را‬
‫ﺣﻞ ﮐﻨﯿﺪ و ﺑﺎ ﺗﻮﺟﻪ ﺑﻪ اﯾﻨﮑﻪ ﺑﺮاي اﮐﺜﺮ ﺳﻮاﻻت ﺑﯿﺶ از ﯾﮏ راه ﺣﻞ ﭘﯿﺸﻨﻬﺎد ﺷﺪه اﺳـﺖ‪ ،‬ﺑـﺎ ﻣﻄﺎﻟﻌـﻪ دﻗﯿـﻖ ﺣـﻞ‬
‫ﺳﻮاﻻت‪ ،‬ﯾﺎدﮔﯿﺮي ﺧﻮد را ﮐﺎﻣﻞ ﮐﻨﯿﺪ‪ .‬ﺑﺎ ﺗﻮﺟﻪ ﺑﻪ اﯾﻨﮑﻪ ﺣﻞ ﺳﻮاﻻت ﻣﺎﻫﯿـﺖ رﯾﺎﺿـﯽ دارد‪ ،‬داﻧﺴـﺘﻦ ﻣﻌﻨـﯽ ﻟﻐـﺎت‬
‫زﯾﺎدي ﺿﺮوري ﻧﯿﺴﺖ و در ﺻﻮرت ﻟﺰوم ﻣﯽﺗﻮاﻧﯿﺪ ﻣـﺜﻼ از ‪ google.com/translate‬ﺑـﺮاي ﺗﺮﺟﻤـﻪ ﻟﻐـﺎت اﺳـﺘﻔﺎده‬
‫ﮐﻨﯿﺪ‪.‬‬
‫ﺑﺎ آرزوي ﺳﻼﻣﺘﯽ ﺑﺮاي ﻫﻤﻪ داﻧﺸﺠﻮﯾﺎن‪ ،‬ﺑﺎﻏﺴﺘﺎﻧﯽ‬
‫ﻣﺜﺎلﻫﺎي ﻓﺼﻞ اول‪ :‬ﻣﻘﺪﻣﻪ و ﻣﻌﺮﻓﯽ درس‬

‫ﻣﺜﺎل ‪) :1-1‬ﺗﺒﺪﯾﻞ واﺣﺪﻫﺎ(‬


‫‪ 1400‬را ﺑﺮﺣﺴﺐ ﻧﯿﻮﺗﻦ ﻣﺤﺎﺳﺒﻪ ﮐﻨﯿﺪ‪ .‬ﺟﺮم ﻣﺎﺷﯿﻦ را ﺑﺮﺣﺴﺐ اﺳﻼگ و ﺳـﭙﺲ وزن را‬ ‫وزن ﻣﺎﺷﯿﻦ ﺑﺎ ﺟﺮم‬
‫ﺑﺮﺣﺴﺐ ﭘﻮﻧﺪ ﺑﯿﺎن ﮐﻨﯿﺪ‪.‬‬

‫ﻣﺜﺎل ‪) :2-1‬ﻗﺎﻧﻮن ﮔﺮاﻧﺶ ﻧﯿﻮﺗﻦ(‬


‫‪ 70‬را ﮐﻪ ﺑـﺮ ﺳـﻄﺢ زﻣـﯿﻦ اﯾﺴـﺘﺎده اﺳـﺖ‪ ،‬ﻣﺤﺎﺳـﺒﻪ‬ ‫ﺑﺎ اﺳﺘﻔﺎده از ﻗﺎﻧﻮن ﮔﺮاﻧﺶ ﻧﯿﻮﺗﻦ وزن ﯾﮏ ﻓﺮد ﺑﻪ ﺟﺮم‬
‫= ﺗﮑﺮار ﮐﻨﯿﺪ و ﻧﺘﯿﺠﻪ را ﻣﻘﺎﯾﺴﻪ ﮐﻨﯿﺪ‪.‬‬ ‫ﮐﻨﯿﺪ‪ .‬ﻣﺤﺎﺳﺒﻪ را ﺑﺎ ﻓﺮﻣﻮل‬

‫ﻣﺜﺎل ‪) :3-1‬ﻋﻤﻠﯿﺎت ﺑﺮداري(‬


‫ﺑﺮاي ﺑﺮدار ‪ v‬و ‪ v‬ﻧﺸﺎن داده ﺷﺪه در ﺷﮑﻞ‪ (a ،‬اﻧﺪازه ﻣﺠﻤﻮع دو ﺑـﺮدار‪ = v + v ،‬را ﺑﯿﺎﺑﯿـﺪ‪ (b .‬زاوﯾـﻪ‬
‫ﺑﺮدار ﻣﺠﻤﻮع ﺑﺎ راﺳﺘﺎي ﻣﺜﺒﺖ ﻣﺤﻮر ‪ x‬را ﺑﯿﺎﺑﯿﺪ‪ (c .‬ﺑﺮدار ﻣﺠﻤﻮع را ﺑﺮﺣﺴﺐ ﺑﺮدارﻫﺎي ﯾﮑﻪ ‪ i‬و ‪ j‬ﺑﻨﻮﯾﺴﯿﺪ و ﺑـﺮدار‬
‫را ﺑﯿﺎﺑﯿﺪ‪.‬‬ ‫ﯾﮑﻪ راﺳﺘﺎي ‪ s‬را ﻣﺤﺎﺳﺒﻪ ﮐﻨﯿﺪ‪ (d .‬ﺑﺮدار ﺗﻔﺎﺿﻞ ‪= v − v‬‬
c01.qxd 10/29/07 1:46 PM Page 19

Article 1/9 Chapter Review 19

Sample Problem 1/1

Determine the weight in newtons of a car whose mass is 1400 kg. Convert
the mass of the car to slugs and then determine its weight in pounds.

Solution. From relationship 1/3, we have


 W  mg  1400(9.81)  13 730 N Ans.
Helpful Hints
From the table of conversion factors inside the front cover of the textbook, we
see that 1 slug is equal to 14.594 kg. Thus, the mass of the car in slugs is  Our calculator indicates a result of
13 734 N. Using the rules of signifi-
cant-figure display used in this text-
 m  1400 kg 14.594
1 slug
kg
 95.9 slugs Ans. book, we round the written result to
four significant figures, or 13 730 N.
Finally, its weight in pounds is Had the number begun with any
digit other than 1, we would have
 W  mg  (95.9)(32.2)  3090 lb Ans. rounded to three significant figures.

As another route to the last result, we can convert from kg to lbm. Again using  A good practice with unit conversion
the table inside the front cover, we have is to multiply by a factor such as

0.45359  1 slug

kg
1 lbm , which has a value of 1,
m  1400 kg  3090 lbm 14.594 kg
because the numerator and the de-
nominator are equivalent. Make sure
The weight in pounds associated with the mass of 3090 lbm is 3090 lb, as calcu- that cancellation of the units leaves
lated above. We recall that 1 lbm is the amount of mass which under standard the units desired; here the units of
conditions has a weight of 1 lb of force. We rarely refer to the U.S. mass unit lbm kg cancel, leaving the desired units
in this textbook series, but rather use the slug for mass. The sole use of slug, of slug.
rather than the unnecessary use of two units for mass, will prove to be powerful
and simple—especially in dynamics.
 Note that we are using a previously calculated result (95.9 slugs). We must be sure that when a calculated number is
needed in subsequent calculations, it is retained in the calculator to its full accuracy, (95.929834 . . .) until it is needed.
This may require storing it in a register upon its initial calculation and recalling it later. We must not merely punch 95.9
into our calculator and proceed to multiply by 32.2—this practice will result in loss of numerical accuracy. Some
individuals like to place a small indication of the storage register used in the right margin of the work paper, directly
beside the number stored.

Sample Problem 1/2


Use Newton’s law of universal gravitation to calculate the weight of a 70-kg
person standing on the surface of the earth. Then repeat the calculation by using
W  mg and compare your two results. Use Table D/2 as needed.

Solution. The two results are


Gmem (6.673  1011)(5.976  1024)(70)
 W   688 N Ans.
R2 [6371  103]
2

W  mg  70(9.81)  687 N Ans.

The discrepancy is due to the fact that Newton’s universal gravitational law does
not take into account the rotation of the earth. On the other hand, the value g 
Helpful Hint
9.81 m/s2 used in the second equation does account for the earth’s rotation. Note
that had we used the more accurate value g  9.80665 m/s2 (which likewise ac-  The effective distance between the
counts for the earth’s rotation) in the second equation, the discrepancy would mass centers of the two bodies in-
have been larger (686 N would have been the result). volved is the radius of the earth.
c01.qxd 10/29/07 1:46 PM Page 20

20 Chapter 1 Introduction to Statics

Sample Problem 1/3

For the vectors V1 and V2 shown in the figure,


(a) determine the magnitude S of their vector sum S  V1  V2
(b) determine the angle  between S and the positive x-axis
(c) write S as a vector in terms of the unit vectors i and j and then write a unit
vector n along the vector sum S
(d) determine the vector difference D  V1  V2

Solution (a) We construct to scale the parallelogram shown in Fig. a for adding
V1 and V2. Using the law of cosines, we have

S2  32  42  2(3)(4) cos 105

S  5.59 units Ans.

 (b) Using the law of sines for the lower triangle, we have

sin 105 sin(  30)



5.59 4

sin(  30)  0.692

(  30)  43.8   13.76 Ans.

(c) With knowledge of both S and , we can write the vector S as

S  S[i cos   j sin ]

 S[i cos 13.76  j sin 13.76]  5.43i  1.328j units Ans.

 Then S 5.43i  1.328j


n   0.971i  0.238j Ans.
S 5.59
(d) The vector difference D is
Helpful Hints
D  V1  V2  4(i cos 45  j sin 45)  3(i cos 30  j sin 30)
 You will frequently use the laws of
 0.230i  4.33j units Ans. cosines and sines in mechanics. See
Art. C/6 of Appendix C for a review of
The vector D is shown in Fig. b as D  V1  (V2). these important geometric principles.

 A unit vector may always be formed


by dividing a vector by its magnitude.
Note that a unit vector is dimen-
sionless.
‫ﻣﺜﺎلﻫﺎي ﻓﺼﻞ دوم‪ :‬ﺳﯿﺴﺘﻢ ﻧﯿﺮوﻫﺎ‬

‫ﻣﺜﺎل ‪ 1-2‬اﻟﯽ ‪ 4-2‬درﺑﺎره ﻋﻤﻠﯿﺎت ﺑﺮداري ﺑﺮ روي ﻧﯿﺮوﻫﺎ در ﺣﺎﻟﺖ ‪ 2‬ﺑﻌﺪي‪.‬‬

‫ﻣﺜﺎل ‪:1-2‬‬
‫در ﻧﻘﻄﻪ ‪ A‬از ﻧﺒﺸﯽ اﺛﺮ ﻣﯽﮐﻨﻨﺪ‪ .‬ﻣﻮﻟﻔﻪ ﻫﺎي ﻣﺴﺘﻄﯿﻠﯽ ﻫﺮ ﯾﮏ از ﻧﯿﺮوﻫﺎ را ﺑﯿﺎﺑﯿﺪ‪.‬‬ ‫و‬ ‫‪،‬‬ ‫ﻧﯿﺮوﻫﺎي‬

‫ﻣﺜﺎل‪:2-2‬‬
‫را ﮐﻪ ﻣﻄﺎﺑﻖ ﺷﮑﻞ ﺑﺮ ﻧﻘﻄﻪ ‪ B‬از ﺳﺎزه اﺛﺮ ﻣﯽﮐﻨﻨﺪ‪ ،‬ﺑﺎ ﯾﮏ ﻧﯿﺮوي ﻣﻌﺎدل ﺟﺎﯾﮕﺰﯾﻦ ﮐﻨﯿﺪ‪.‬‬ ‫و‬ ‫دو ﻧﯿﺮوي‬

‫ﻣﺜﺎل ‪:3-2‬‬
‫ﻧﯿﺮوي ‪ = 500‬ﻣﻄﺎﺑﻖ ﺷﮑﻞ ﺑﻪ ﻣﯿﻠﻪ ﻋﻤـﻮدي وارد ﻣـﯽﺷـﻮد‪ (1 .‬ﻧﯿـﺮوي را ﺑﺮﺣﺴـﺐ ﺑـﺮدار ﯾﮑـﻪ ‪ i‬و‪j‬‬
‫ﺑﻨﻮﯾﺴﯿﺪ و ﻣﻮﻟﻔﻪ ﻫﺎي ﻣﺴﺘﻄﯿﻠﯽ آن را ﺗﻌﯿﯿﻦ ﮐﻨﯿﺪ‪ (2 .‬ﻣﻮﻟﻔﻪ ﻫﺎي ﻋﺪدي ﺑﺮدار را در راﺳﺘﺎي و ﭘﯿﺪا ﮐﻨﯿﺪ‪.‬‬
‫‪ (3‬ﻣﻮﻟﻔﻪ ﻫﺎي ﻋﺪدي ﺑﺮدار را در راﺳﺘﺎي و ﭘﯿﺪا ﮐﻨﯿﺪ‪.‬‬

‫ﻣﻄﺎﺑﻖ ﺷﮑﻞ ﺑﺮ ﻧﺒﺸﯽ اﺛﺮ ﻣﯽﮐﻨﻨﺪ‪ .‬ﺗﺼﻮﯾﺮ ﻧﯿﺮوي ﺑﺮآﯾﻨﺪ آﻧﻬﺎ را در راﺳﺘﺎي ‪ b‬ﺑﯿﺎﺑﯿﺪ‪.‬‬ ‫و‬ ‫ﻣﺜﺎل ‪ :4-2‬ﻧﯿﺮوي‬
c02.qxd 10/29/07 1:38 PM Page 29

Article 2/3 Rectangular Components 29

Sample Problem 2/1 y


F2 = 500 N F1 = 600 N
The forces F1, F2, and F3, all of which act on point A of the bracket, are
specified in three different ways. Determine the x and y scalar components of
each of the three forces. 3 A 35°
4
0.1 m
x
Solution. The scalar components of F1, from Fig. a, are
0.2 m

F1x  600 cos 35  491 N Ans. 0.3 m

F1y  600 sin 35  344 N Ans. F3 = 800 N


B
The scalar components of F2, from Fig. b, are 0.4 m


4
F2x  500 5  400 N Ans.

F1 = 600 N

3
F2y  500 5  300 N Ans.
F1 y

A
F3 x
Note that the angle which orients F2 to the x-axis is never calculated. The cosine 35°

F3
and sine of the angle are available by inspection of the 3-4-5 triangle. Also note A F1 x

=8
that the x scalar component of F2 is negative by inspection. (a) α

00
The scalar components of F3 can be obtained by first computing the angle  0.4 m

N
of Fig. c.
F2 = 500 N F3 y
  tan1 0.2
0.4
 26.6
3
F2 y

0.2 m B
4
 Then F3x  F3 sin   800 sin 26.6  358 N Ans. F2 x A (c)

F3y  F3 cos   800 cos 26.6  716 N Ans. (b)

Alternatively, the scalar components of F3 can be obtained by writing F3 as Helpful Hints


a magnitude times a unit vector nAB in the direction of the line segment AB.
Thus,  You should carefully examine the
geometry of each component deter-

 
AB
l
0.2i  0.4j mination problem and not rely on
 F3  F3nAB  F3   800 the blind use of such formulas as
AB (0.2)2  (0.4)2
Fx  F cos  and Fy  F sin .
 800 [0.447i  0.894j]
 A unit vector can be formed by di-
 358i  716j N
viding any vector, such as the geo-
l
The required scalar components are then metric position vector AB , by its
length or magnitude. Here we use
F3x  358 N Ans. the overarrow to denote the vector
which runs from A to B and the
F3y  716 N Ans. overbar to determine the distance
between A and B.
which agree with our previous results.
c02.qxd 10/29/07 1:38 PM Page 30

30 Chapter 2 Force Systems

Sample Problem 2/2 B


P = 800 N
N
Combine the two forces P and T, which act on the fixed structure at B, into 0
60 y
a single equivalent force R. =
T
6m x

Graphical solution. The parallelogram for the vector addition of forces T and A α C 60° D
 P is constructed as shown in Fig. a. The scale used here is 1 cm  400 N; a scale
of 1 cm  100 N would be more suitable for regular-size paper and would give 3m
greater accuracy. Note that the angle a must be determined prior to construction
of the parallelogram. From the given figure

BD 6 sin 60
tan     0.866   40.9 B 800 N
AD 3  6 cos 60 P
N θ
0
Measurement of the length R and direction  of the resultant force R yields the 60
approximate results α
T R
(a)
R  525 N   49 Ans.

Geometric solution. The triangle for the vector addition of T and P is shown Helpful Hints
 in Fig. b. The angle  is calculated as above. The law of cosines gives  Note the repositioning of P to per-
mit parallelogram addition at B.
R2  (600)2  (800)2  2(600)(800) cos 40.9  274,300

R  524 N Ans.
800 N
B P
From the law of sines, we may determine the angle  which orients R. Thus, θ α
R 600 N
600 524 α
 sin   0.750   48.6 Ans.
sin  sin 40.9 T
(b)

Algebraic solution. By using the x-y coordinate system on the given figure,
we may write
 Note the repositioning of F so as to
Rx  ΣFx  800  600 cos 40.9  346 N preserve the correct line of action of
the resultant R.
Ry  ΣFy  600 sin 40.9  393 N

The magnitude and dipection of the resultant force R as shown in Fig. c are then y

R  Rx2  Ry2  (346)2  (393)2  524 N Ans. Rx = 346 N


B x
Ry  393 θ
  tan1  tan1  48.6 Ans.
Rx  346 Ry = – 393 N

The resultant R may also be written in vector notation as R


(c)
R  Rxi  Ry j  346i  393j N Ans.
c02.qxd 10/29/07 1:38 PM Page 31

Article 2/3 Rectangular Components 31

Sample Problem 2/3 y

The 500-N force F is applied to the vertical pole as shown. (1) Write F in j
terms of the unit vectors i and j and identify both its vector and scalar compo-
nents. (2) Determine the scalar components of the force vector F along the j′ y′
x- and y-axes. (3) Determine the scalar components of F along the x- and y-axes. A 30°
x
i

F = 500 N
Solution. Part (1). From Fig. a we may write F as 30°

F  (F cos )i  (F sin )j i′

 (500 cos 60)i  (500 sin 60)j x′

 (250i  433j) N Ans. y y′

The scalar components are Fx  250 N and Fy  433 N. The vector compo- Fx
A x A j′
nents are Fx  250i N and Fy  433j N. θ = 60°
Part (2). From Fig. b we may write F as F  500i N, so that the required F
scalar components are Fy F
i′
Fx  500 N Fy  0 Ans.

Part (3). The components of F in the x- and y-directions are nonrectan- (a) (b) x′
gular and are obtained by completing the parallelogram as shown in Fig. c. The
y′ Fx
magnitudes of the components may be calculated by the law of sines. Thus,
x
60° 30°
Fx 500 90°
  Fx  1000 N Fy′
90°
sin 90 sin 30 30° 60°
Fy F = 500 N
500
 Fy  866 N (c)
sin 60 sin 30

The required scalar components are then Helpful Hint


 Obtain Fx and Fy graphically and
Fx  1000 N Fy  866 N Ans.
compare your results with the calcu-
lated values.

a
Sample Problem 2/4 F1 = 100 N

Forces F1 and F2 act on the bracket as shown. Determine the projection Fb C


of their resultant R onto the b-axis. 30°
20°

F2 = 80 N b

Solution. The parallelogram addition of F1 and F2 is shown in the figure.


Using the law of cosines gives us a
F1
R2  (80)2  (100)2  2(80)(100) cos 130 R  163.4 N
0N
10 R
The figure also shows the orthogonal projection Fb of R onto the b-axis. Its C 50°
length is 80 N

Fb  80  100 cos 50  144.3 N Ans. F2 50°


Fb
Note that the components of a vector are in general not equal to the projec-
tions of the vector onto the same axes. If the a-axis had been perpendicular to b
the b-axis, then the projections and components of R would have been equal.
‫ﻣﺜﺎل ‪ 5-2‬و ‪ 6-2‬درﺑﺎره ﮔﺸﺘﺎور ﻧﯿﺮوﻫﺎ در ﺣﺎﻟﺖ ‪ 2‬ﺑﻌﺪي‪.‬‬

‫ﻣﺜﺎل ‪:5-2‬‬
‫‪ 600‬را ﺣﻮل ﻧﻘﻄﻪ ‪ O‬از ﭘﺎﯾﻪ ﻣﯿﻠﻪ‪ ،‬ﺑﺎ روشﻫﺎي ﻣﺨﺘﻠﻒ ﻣﺤﺎﺳﺒﻪ ﮐﻨﯿﺪ‪.‬‬ ‫ﮔﺸﺘﺎور ﻧﯿﺮوي‬

‫ﻣﺜﺎل ‪:6-2‬‬
‫ﺗﻠﻪ در ‪ OA‬ﺑﺎ ﮐﺎﺑﻞ ‪ AB‬ﮐﺸﯿﺪه ﻣﯽﺷﻮد ﮐﻪ از ﭘﻮﻟﯽ ﻫﺪاﯾﺖ ﮐﻨﻨﺪه ﺑﺪون اﺻـﻄﮑﺎك در ‪ B‬ﻣـﯽﮔـﺬرد‪ .‬ﮐﺸـﺶ در‬
‫ﺑﺮﺣﺴـﺐ‬ ‫ﺣﻮل ﻟﻮﻻي ‪ O‬ﻣـﯽﺷـﻮد‪ .‬ﮐﻤﯿـﺖ ‪⁄‬‬ ‫ﮐﺎﺑﻞ ﻫﻤﻪ ﺟﺎ ﺑﺮاﺑﺮ ‪ T‬اﺳﺖ و اﯾﻦ ﮐﺸﺶ ﺳﺒﺐ ﮔﺸﺘﺎور‬
‫زاوﯾﻪ ﺑﺎزﺷﺪن در ‪ ، θ‬ﺑﺮاي ‪ 0 ≤ θ ≤ 90‬رﺳﻢ ﮐﺮده و ﺣﺪاﻗﻞ و ﺣﺪاﮐﺜﺮ ﻣﻘﺪار آن را ﺑﯿﺎﺑﯿﺪ‪ .‬اﻫﻤﯿﺖ ﻓﯿﺰﯾﮑﯽ اﯾـﻦ‬
‫ﮐﻤﯿﺖ ﭼﯿﺴﺖ؟‬
c02.qxd 10/29/07 1:38 PM Page 41

Article 2/4 Moment 41

Sample Problem 2/5 2m


A
Calculate the magnitude of the moment about the base point O of the 600-N
force in five different ways. 40°

4m 600 N

Solution. (I) The moment arm to the 600-N force is

O
d  4 cos 40  2 sin 40  4.35 m
2m
 By M  Fd the moment is clockwise and has the magnitude
40°
MO  600(4.35)  2610 N  m Ans.
4m
600 N
(II) Replace the force by its rectangular components at A 40° d

F1  600 cos 40  460 N, F2  600 sin 40  386 N O

By Varignon’s theorem, the moment becomes 2m F1 = 600 cos 40°

 MO  460(4)  386(2)  2610 N  m Ans.


4m
F2 = 600 sin 40°
(III) By the principle of transmissibility, move the 600-N force along its
line of action to point B, which eliminates the moment of the component F2. The
moment arm of F1 becomes
O
d1  4  2 tan 40  5.68 m B
F1 y

and the moment is F2


A
x
MO  460(5.68)  2610 N  m Ans.
d1 r F
 (IV) Moving the force to point C eliminates the moment of the component
F1. The moment arm of F2 becomes
C
F1
d2  2  4 cot 40  6.77 m O d2

and the moment is F2

MO  386(6.77)  2610 N  m Ans.


Helpful Hints
(V) By the vector expression for a moment, and by using the coordinate  The required geometry here and in
system indicated on the figure together with the procedures for evaluating cross similar problems should not cause dif-
products, we have ficulty if the sketch is carefully drawn.
 This procedure is frequently the
 MO  r  F  (2i  4j)  600(i cos 40  j sin 40) shortest approach.
 2610k N  m  The fact that points B and C are not
on the body proper should not cause
The minus sign indicates that the vector is in the negative z-direction. The mag- concern, as the mathematical calcula-
nitude of the vector expression is tion of the moment of a force does not
require that the force be on the body.
MO  2610 N  m Ans.  Alternative choices for the position
vector r are r  d1 j  5.68j m and
r  d2i  6.77i m.
c02.qxd 10/29/07 1:38 PM Page 42

42 Chapter 2 Force Systems

Sample Problem 2/6 B


T
The trap door OA is raised by the cable AB, which passes over the small fric- 0.3 m
tionless guide pulleys at B. The tension everywhere in the cable is T, and this ten- 0.4 m A
sion applied at A causes a moment MO about the hinge at O. Plot the quantity MO/T 0.5 m
as a function of the door elevation angle  over the range 0    90 and note min- O
θ
imum and maximum values. What is the physical significance of this ratio?

Solution. We begin by constructing a figure which shows the tension force T y


acting directly on the door, which is shown in an arbitrary angular position . It
should be clear that the direction of T will vary as  varies. In order to deal with B
rAB
this variation, we write a unit vector nAB which “aims” T: T
rAB rOB  rOA rOB A
 nAB  r  rAB d
AB rOA

Using the x-y coordinates of our figure, we can write O θ


x
 rOB  0.4j m and rOA  0.5(cos i  sin j) m
So Helpful Hints
rAB  rOB  rOA  0.4j  (0.5)(cos i  sin j)  Recall that any unit vector can be
 0.5 cos i  (0.4  0.5 sin )j m written as a vector divided by its
magnitude. In this case the vector in
and
the numerator is a position vector.
rAB  (0.5 cos )2  (0.4  0.5 sin )2
0.5
 0.41  0.4 sin  m
0.4
The desired unit vector is
0.3
rAB 0.5 cos i  (0.4  0.5 sin )j MO
nAB  r  ——– ,
T
m
AB 0.41  0.4 sin  0.2

Our tension vector can now be written as 0.1

T  TnAB  T 0.5 cos0.41


i  (0.4  0.5 sin )j
 0.4 sin 
 0
0 10 20 30 40 50 60 70 80 90
θ , deg
 The moment of T about point O, as a vector, is MO  rOB  T, where rOB  0.4j m, or
 Recall that any vector may be writ-
MO  0.4j  T 0.5 cos0.41
i  (0.4  0.5 sin )j
 0.4 sin 
 ten as a magnitude times an “aim-
ing” unit vector.
0.2T cos 
 k
0.41  0.4 sin   In the expression M  r  F, the po-
The magnitude of MO is sition vector r runs from the mo-
ment center to any point on the line
0.2T cos 
MO  of action of F. Here, rOB is more con-
0.41  0.4 sin  venient than rOA.
and the requested ratio is
MO 0.2 cos 
 Ans.
T 0.41  0.4 sin 
which is plotted in the accompanying graph. The expression MO/T is the moment
arm d (in meters) which runs from O to the line of action of T. It has a maximum
value of 0.4 m at   53.1 (at which point T is horizontal) and a minimum value of
0 at   90 (at which point T is vertical). The expression is valid even if T varies.
This sample problem treats moments in two-dimensional force systems, and
it also points out the advantages of carrying out a solution for an arbitrary posi-
tion, so that behavior over a range of positions can be examined.
‫ﻣﺜﺎل ‪ 7-2‬و ‪ 8-2‬درﺑﺎره ﮐﻮﭘﻞ ﯾﺎ زوج ﻧﯿﺮو‪.‬‬

‫ﻣﺜﺎل ‪:7-2‬‬
‫و‬ ‫‪ 100‬ﻗﺮار دارد‪ .‬اﯾﻦ ﮐﻮﭘﻞ را ﺑﺎ ﮐﻮﭘﻞ ﻣﻌﺎدل دو ﻧﯿـﺮوي‬ ‫ﺳﺎزه ﺻﻠﺐ ﻧﺸﺎن داده ﺷﺪه ﺗﺤﺖ ﮐﻮﭘﻞ دو ﻧﯿﺮوي‬
‫‪ 400‬ﺟﺎﯾﮕﺰﯾﻦ ﮐﻨﯿﺪ‪ .‬زاوﯾﻪ درﺳﺖ ‪ θ‬را ﻣﺸﺨﺺ ﮐﻨﯿﺪ‪.‬‬ ‫– ‪ ،‬ﻫﺮ ﯾﮏ ﺑﺎ اﻧﺪازه‬

‫‪ 400‬وارد ﺑﺮ اﻫﺮم را ﺑﺎ ﯾﮏ ﻧﯿﺮو و ﮐﻮﭘﻞ ﻣﻌﺎدل در ‪ O‬ﺟﺎﯾﮕﺰﯾﻦ ﮐﻨﯿﺪ‪.‬‬ ‫ﻣﺜﺎل ‪ :8-2‬ﻧﯿﺮوي اﻓﻘﯽ‬
c02.qxd 10/29/07 1:38 PM Page 52

52 Chapter 2 Force Systems

Sample Problem 2/7


M
The rigid structural member is subjected to a couple consisting of the two
100-N forces. Replace this couple by an equivalent couple consisting of the two 40 P
θ
forces P and P, each of which has a magnitude of 400 N. Determine the proper θ
angle . –P

100
Solution. The original couple is counterclockwise when the plane of the forces
is viewed from above, and its magnitude is
100
[M  Fd] M  100(0.1)  10 N  m
The forces P and P produce a counterclockwise couple
60
100
M  400(0.040) cos 
100 N

 Equating the two expressions gives 100 N

10  (400)(0.040) cos  Dimensions in millimeters

10
  cos1 16  51.3 Ans. P = 400 N
θ
Helpful Hint
40 mm

 Since the two equal couples are parallel free vectors, the only dimensions
which are relevant are those which give the perpendicular distances between θ
the forces of the couples. P = 400 N

Sample Problem 2/8


400 N
Replace the horizontal 400-N force acting on the lever by an equivalent sys-
tem consisting of a force at O and a couple.
200 mm

60°
O
Solution. We apply two equal and opposite 400-N forces at O and identify the
counterclockwise couple

[M  Fd] M  400(0.200 sin 60)  69.3 N  m Ans. 400 N 400 N

 Thus, the original force is equivalent to the 400-N force at O and the 69.3- N  m
couple as shown in the third of the three equivalent figures.
≡ ≡
Helpful Hint
O O O
 The reverse of this problem is often encountered, namely, the replacement
400 N 400 N 400 N
of a force and a couple by a single force. Proceeding in reverse is the same as
69.3 N.m
replacing the couple by two forces, one of which is equal and opposite to the
400-N force at O. The moment arm to the second force would be M/F 
69.3/400  0.1732 m, which is 0.2 sin 60, thus determining the line of ac-
tion of the single resultant force of 400 N.
‫ﻣﺜﺎل ‪) :9-2‬ﺑﺮآﯾﻨﺪ ﻧﯿﺮوﻫﺎ(‬
‫ﺑﺮآﯾﻨﺪ ‪ 4‬ﻧﯿﺮو و ﯾﮏ ﮐﻮﭘﻞ وارده ﺑﺮ ورق ﻧﺸﺎن داده ﺷﺪه را ﺑﯿﺎﺑﯿﺪ‪.‬‬
c02.qxd 10/29/07 1:38 PM Page 60

60 Chapter 2 Force Systems

Sample Problem 2/9 y


2m
Determine the resultant of the four forces and one couple which act on the 60 N 5m
plate shown. 50 N

45°
Solution. Point O is selected as a convenient reference point for the force–couple 140 N·m
2m
system which is to represent the given system.
80 N
[Rx  ΣFx] Rx  40  80 cos 30  60 cos 45  66.9 N 2m
40 N O 30° x
[Ry  ΣFy] Ry  50  80 sin 30  60 cos 45  132.4 N 1m

[R  Rx2  Ry2] R  (66.9)2  (132.4)2  148.3 N Ans.


y

  tan 
Ry 132.4
1   tan1  63.2 Ans.
Rx 66.9 R = 148.3 N
 [MO  Σ(Fd)] MO  140  50(5)  60 cos 45(4)  60 sin 45(7)
(a)
θ = 63.2°
 237 N  m |MO| =
237 N·m x
O
The force–couple system consisting of R and MO is shown in Fig. a.
We now determine the final line of action of R such that R alone represents
the original system.
R = 148.3 N
[Rd  MO] 148.3d  237 d  1.600 m Ans.
Hence, the resultant R may be applied at any point on the line which makes a (b) 1.600 m 63.2°
63.2 angle with the x-axis and is tangent at point A to a circle of 1.600-m radius A
with center O, as shown in part b of the figure. We apply the equation Rd  MO in x
O
an absolute-value sense (ignoring any sign of MO) and let the physics of the situa- B
tion, as depicted in Fig. a, dictate the final placement of R. Had MO been counter-
clockwise, the correct line of action of R would have been the tangent at point B.
The resultant R may also be located by determining its intercept distance b 132.4 x – 66.9 y =
to point C on the x-axis, Fig. c. With Rx and Ry acting through point C, only Ry –237 y
(c)
exerts a moment about O so that
C
x
237 O
Ry b  MO and b  1.792 m
132.4 R b
Alternatively, the y-intercept could have been obtained by noting that the mo-
ment about O would be due to Rx only. Helpful Hints
A more formal approach in determining the final line of action of R is to use
 We note that the choice of point O as
the vector expression
a moment center eliminates any mo-
r  R  MO ments due to the two forces which
pass through O. Had the clockwise
where r  xi  yj is a position vector running from point O to any point on the sign convention been adopted, MO
line of action of R. Substituting the vector expressions for r, R, and MO and car- would have been 237 N  m, with
rying out the cross product result in the plus sign indicating a sense
(xi  yj)  (66.9i  132.4j)  237k which agrees with the sign conven-
tion. Either sign convention, of
(132.4x  66.9y)k  237k course, leads to the conclusion of a
clockwise moment MO.
Thus, the desired line of action, Fig. c, is given by
 Note that the vector approach
132.4x  66.9y  237 yields sign information automati-
cally, whereas the scalar approach
 By setting y  0, we obtain x  1.792 m, which agrees with our earlier calcula- is more physically oriented. You
tion of the distance b. should master both methods.
‫ﻣﺜﺎل ‪) :10-2‬ﻣﻮﻟﻔﻪ ﻫﺎي ﯾﮏ ﻧﯿﺮو در ﺣﺎﻟﺖ ﺳﻪ ﺑﻌﺪي(‬
‫ﯾﮏ ﻧﯿﺮوي ‪ F‬ﺑﺎ ﺑﺰرﮔﯽ ‪ 100‬در ﻧﻘﻄﻪ ‪ o‬از دﺳﺘﮕﺎه ﻣﺨﺘﺼﺎت ﻧﺸﺎن داده ﺷﺪه وارد ﻣﯽﺷـﻮد‪ .‬ﺧـﻂ اﺛـﺮ ﻧﯿـﺮو از‬
‫ﻧﻘﻄﻪ ‪ A‬ﺑﺎ ﻣﺨﺘﺼﺎت )‪ (3,4,5‬ﻣﯽﮔﺬرد‪ (1 .‬ﻣﻮﻟﻔﻪ ﻫﺎي اﺳﮑﺎﻟﺮ ‪ F‬در راﺳﺘﺎي ‪ x‬و ‪ y‬و ‪ z‬ﺑﯿﺎﺑﯿـﺪ‪ (2 .‬ﺗﺼـﻮﯾﺮ ‪ F‬ﺑـﺮ‬
‫ﺻﻔﺤﻪ ‪ xy‬را ﺑﯿﺎﺑﯿﺪ‪ (3 .‬ﺗﺼﻮﯾﺮ ‪ F‬در راﺳﺘﺎي ﺧﻂ ‪ OB‬ﺑﯿﺎﺑﯿﺪ‪.‬‬
c02.qxd 10/29/07 1:38 PM Page 69

Article 2/7 Rectangular Components 69

Sample Problem 2/10 F = 100 N


z
A force F with a magnitude of 100 N is applied at the origin O of the axes
4m
x-y-z as shown. The line of action of F passes through a point A whose coordi- A
nates are 3 m, 4 m, and 5 m. Determine (a) the x, y, and z scalar components of
F, (b) the projection Fxy of F on the x-y plane, and (c) the projection FOB of F
along the line OB. y B
5m
2m
6m
Solution. Part (a). We begin by writing the force vector F as its magnitude
F times a unit vector nOA. O 3m 6m

 
l
OA 3i  4j  5k
F  FnOA  F  100 x
OA 32  42  52
z
 100[0.424i  0.566j  0.707k]
F
 42.4i  56.6j  70.7k N
The desired scalar components are thus Fz
y
 Fx  42.4 N Fy  56.6 N Fz  70.7 N Ans.

Part (b). The cosine of the angle xy between F and the x-y plane is Fy

32  42 θ xy Fxy = 70.7 N


cos xy   0.707
32  42  52
O
so that Fxy  F cos xy  100(0.707)  70.7 N Ans. Fx

Part (c). The unit vector nOB along OB is x


l
OB 6i  6j  2k
nOB    0.688i  0.688j  0.229k
OB 62  62  22 F

The scalar projection of F on OB is

 FOB  F  nOB  (42.4i  56.6j  70.7k)  (0.688i  0.688j  0.229k)


z
 (42.4)(0.688)  (56.6)(0.688)  (70.7)(0.229)
nOB
 84.4N Ans. y
FOB = 84.4 N
If we wish to express the projection as a vector, we write
O
FOB  F  nOBnOB
x
 84.4(0.688i  0.688j  0.229k)
Helpful Hints
 58.1i  58.1j  19.35k N
 In this example all scalar components
are positive. Be prepared for the case
where a direction cosine, and hence
the scalar component, are negative.
 The dot product automatically finds
the projection or scalar component
of F along line OB as shown.
‫ﻣﺜﺎلﻫﺎي ‪ 11-2‬ﺗﺎ ‪ 15-2‬درﺑﺎره ﮔﺸﺘﺎور و ﮐﻮﭘﻞ ﻧﯿﺮوﻫﺎ در ﺣﺎﻟﺖ ﺳﻪ ﺑﻌﺪي‬

‫ﻣﺜﺎل ‪:11-2‬‬
‫ﮔﺸﺘﺎور ﻧﯿﺮوي ‪ F‬را ﺣﻮل ﻧﻘﻄﻪ ‪ (1 ،o‬ﺑﺎ ﺷﻬﻮد ﻫﻨﺪﺳﯽ ‪ (2‬ﺑﺎ ﺗﻌﺮﯾﻒ ﺑﺮداري‪ ،‬ﻣﺤﺎﺳﺒﻪ ﮐﻨﯿﺪ‪.‬‬

‫ﻣﺜﺎل ‪:12-2‬‬
‫‪ 2.4‬در ﮐﺎﺑﻞ ‪ AB‬ﻣﺤﮑﻢ ﺷﺪه اﺳﺖ‪ .‬ﮔﺸﺘﺎور و اﻧـﺪازه ﻧﯿـﺮوي ﮐﺸـﺶ در ﮐﺎﺑـﻞ وارد در‬ ‫ﭘﯿﭻ ﺗﺎ اﯾﺠﺎد ﻧﯿﺮوي‬
‫ﻧﻘﻄﻪ ‪ A‬را ﺣﻮل ﻧﻘﻄﻪ ‪ O‬ﻣﺤﺎﺳﺒﻪ ﮐﻨﯿﺪ‪.‬‬

‫ﻣﺜﺎل ‪:13-2‬‬
‫‪ 10‬ﺑﻪ ﺑﺎﻻي ﺳﺘﻮن ﻣﺤﮑﻢ ﺷﺪه اﺳﺖ و ﺑﻪ ﻧﻘﻄﻪ ‪ B‬ﻣﺤﮑﻢ ﺷﺪه اﺳﺖ‪ .‬ﮔﺸـﺘﺎور ﻧﯿـﺮو‬ ‫ﮐﺸﺶ در ﮐﺎﺑﻞ ﺑﻪ اﻧﺪازه‬
‫را ﺣﻮل ﻣﺤﻮر ‪ z‬ﮔﺬرﻧﺪه از ﻧﻘﻄﻪ ‪ O‬ﺑﯿﺎﺑﯿﺪ‪.‬‬

‫ﻣﺜﺎل ‪:14-2‬‬
‫اﻧﺪازه و ﺟﻬﺖ ﮐﻮﭘﻞ ‪ M‬ﻣﻌﺎدل ﺑﺎ دو ﮐﻮﭘﻞ ﻧﺸﺎن داده در ﺷﮑﻞ را ﺑﯿﺎﺑﯿﺪ‪ .‬اﯾﻦ ﮐﻮﭘﻞ ﻣﻌﺎدل را ﺑﺎ دو ﻧﯿﺮوي ‪ F‬و ‪–F‬‬
‫ﮐﻪ ﺑﺮ روي دو وﺟﻪ ﻣﻮازي ‪ y-z‬از ﺑﻠﻮك وارد ﻣﯽﺷﻮد‪ ،‬ﺟﺎﯾﮕﺰﯾﻦ ﮐﻨﯿﺪ‪.‬‬

‫ﻣﺜﺎل ‪:15-2‬‬
‫ﻧﯿﺮوي ‪ 400‬ﺑﻪ ﻧﻘﻄﻪ ‪ A‬از اﻫﺮم ﮐﻨﺘﺮل وارد ﻣﯽﺷﻮد‪ .‬اﯾﻦ ﻧﯿﺮو را ﺑﺎ ﻧﯿﺮو و ﮐﻮﭘﻞ ﻣﻌﺎدل در ‪ O‬ﺟﺎﯾﮕﺰﯾﻦ ﮐﻨﯿـﺪ‪.‬‬
‫ﮐﻮﭘﻞ را ﺑﻪ ﺻﻮرت ﺑﺮداري ﺑﯿﺎن ﮐﻨﯿﺪ‪.‬‬
c02.qxd 10/29/07 1:38 PM Page 78

78 Chapter 2 Force Systems

Sample Problem 2/11 z

Determine the moment of force F about point O (a) by inspection and (b) by b
the formal cross-product definition MO  r  F. a

Solution. (a) Because F is parallel to the y-axis, F has no moment about that c O
axis. It should be clear that the moment arm from the x-axis to the line of action y
of F is c and that the moment of F about the x-axis is negative. Similarly, the
moment arm from the z-axis to the line of action of F is a and the moment of F
about the z-axis is positive. So we have x

MO  cFi  aFk  F(ci  ak) Ans. z

(b) Formally, b
a

 MO  r  F  (ai  ck)  Fj  aFk  cFi F

 F(ci  ak) Ans. r


c O
y
Helpful Hint
 Again we stress that r runs from the moment center to the line of action of F.
Another permissible, but less convenient, position vector is r  ai  bj  ck. x

z
Sample Problem 2/12 1.6 m

The turnbuckle is tightened until the tension in cable AB is 2.4 kN. Deter-
A
mine the moment about point O of the cable force acting on point A and the
magnitude of this moment.

2m

Solution. We begin by writing the described force as a vector. O


y
x
T  TnAB  2.4 0.8  1.5  2 
0.8i  1.5j  2k
2 2 2
1.5 m
B
0.8 m

 0.731i  1.371j  1.829k kN z

1.6 m
The moment of this force about point O is
A
 MO  rOA  T  (1.6i  2k)  (0.731i  1.371j  1.829k)

 2.74i  4.39j  2.19k kN  m Ans. rOA

This vector has a magnitude 2m T

O
MO  2.742  4.392  2.192  5.62 kN  m Ans.
y
x
Helpful Hint 1.5 m
0.8 m
 The student should verify by inspection the signs of the moment components. B
c02.qxd 10/29/07 1:38 PM Page 79

Article 2/8 Moment and Couple 79

Sample Problem 2/13 y

A tension T of magnitude 10 kN is applied to the cable attached to the top A


of the rigid mast and secured to the ground at B. Determine the moment Mz of T A
about the z-axis passing through the base O.

15 m
Solution (a). The required moment may be obtained by finding the compo- T = 10 kN
nent along the z-axis of the moment MO of T about point O. The vector MO is
normal to the plane defined by T and point O, as shown in the accompanying fig-
O
ure. In the use of Eq. 2/14 to find MO, the vector r is any vector from point O to
 the line of action of T. The simplest choice is the vector from O to A, which is x
written as r  15j m. The vector expression for T is z 12 m 9m
B
T  TnAB  10 (12)  (15)  (9) 
12i  15j  9k
2 2 2
Helpful Hints

 10(0.566i  0.707j  0.424k) kN  We could also use the vector from O


From Eq. 2/14, to B for r and obtain the same result,
but using vector OA is simpler.
[MO  r  F] MO  15j  10(0.566i  0.707j  0.424k)
 It is always helpful to accompany your
 150(0.566k  0.424i) kN  m vector operations with a sketch of the
The value Mz of the desired moment is the scalar component of MO in the vectors so as to retain a clear picture
z-direction or Mz  MO  k. Therefore, of the geometry of the problem.

Mz  150(0.566k  0.424i)  k  84.9 kN  m Ans.  Sketch the x-y view of the problem
and show d.
 The minus sign indicates that the vector Mz is in the negative z-direction. Ex-
pressed as a vector, the moment is Mz  84.9k kN  m. y

A
Solution (b). The force of magnitude T is resolved into components Tz and Txy
in the x-y plane. Since Tz is parallel to the z-axis, it can exert no moment about
 this axis. The moment Mz is, then, due only to Txy and is Mz  Txyd, where d is r Mz
the perpendicular distance from Txy to O. The cosine of the angle between T and T Mo
Txy is 152  122 / 152  122  92  0.906, and therefore,
O
Txy  10(0.906)  9.06 kN
x
The moment arm d equals OA multiplied by the sine of the angle between Txy z
and OA, or B

12
d  15  9.37 m
122  152 y

Hence, the moment of T about the z-axis has the magnitude Tz A Tx

Mz  9.06(9.37)  84.9 kN  m Ans.

and is clockwise when viewed in the x-y plane. Ty


T Txy 15 m

Solution (c). The component Txy is further resolved into its components Tx and Ty. O
It is clear that Ty exerts no moment about the z-axis since it passes through it, so
that the required moment is due to Tx alone. The direction cosine of T with respect x
to the x-axis is 12/92  122  152  0.566 so that Tx  10(0.566)  5.66 kN. Thus, z
12 m 9m
B
Mz  5.66(15)  84.9 kN  m Ans.
c02.qxd 10/29/07 1:38 PM Page 80

80 Chapter 2 Force Systems

Sample Problem 2/14 30 N


30 N
Determine the magnitude and direction of the couple M which will replace 60°
the two given couples and still produce the same external effect on the block. mm 60°
Specify the two forces F and F, applied in the two faces of the block parallel to 100 mm 60
x
mm y
the y-z plane, which may replace the four given forces. The 30-N forces act paral- 40
lel to the y-z plane.
25 N
50 mm
Solution. The couple due to the 30-N forces has the magnitude M1  30(0.06) 
1.80 N  m. The direction of M1 is normal to the plane defined by the two forces,
and the sense, shown in the figure, is established by the right-hand convention. 25 N
The couple due to the 25-N forces has the magnitude M2  25(0.10)  2.50 N  m
z
with the direction and sense shown in the same figure. The two couple vectors
combine to give the components M2 = 2.5 N·m

My  1.80 sin 60  1.559 N  m M


x y
Mz  2.50  1.80 cos 60  1.600 N  m θ
–F
 Thus, M  (1.559)2  (1.600)2  2.23 N  m Ans. θ
y θ
1.559
with   tan1 1.600  tan1 0.974  44.3 Ans. 60° F
M1 = 1.8 N·m
The forces F and F lie in a plane normal to the couple M, and their mo-
z
ment arm as seen from the right-hand figure is 100 mm. Thus, each force has the z
magnitude
Helpful Hint

[M = Fd]
2.23
F  0.10  22.3 N Ans.  Bear in mind that the couple vectors
are free vectors and therefore have
and the direction   44.3. no unique lines of action.

Sample Problem 2/15 z

A force of 400 N is applied at A to the handle of the control lever which is at- 400 N
tached to the fixed shaft OB. In determining the effect of the force on the shaft 200 mm
at a cross section such as that at O, we may replace the force by an equivalent 50 mm
force at O and a couple. Describe this couple as a vector M. 75 mm A
O
Solution. The couple may be expressed in vector notation as M  r  F, y
l
where r  OA  0.2j  0.125k m and F  400i N. Thus, x
B
M  (0.2j  0.125k)  (400i)  50j  80k N  m
z
Alternatively we see that moving the 400-N force through a distance d  (400 N)
M
0.1252  0.22  0.236 m to a parallel position through O requires the addition 400 N
of a couple M whose magnitude is
θ A
d
M  Fd  400(0.236)  94.3 N  m Ans. θ
O 125 mm
The couple vector is perpendicular to the plane in which the force is shifted, and 200 mm
x
its sense is that of the moment of the given force about O. The direction of M in y
the y-z plane is given by
125
  tan1 200  32.0 Ans.
‫ﻣﺜﺎلﻫﺎي ‪ 16-2‬ﺗﺎ ‪ 19-2‬درﺑﺎره ﺑﺮآﯾﻨﺪ ﻧﯿﺮوﻫﺎ در ﺣﺎﻟﺖ ﺳﻪ ﺑﻌﺪي‬

‫ﻣﺜﺎل ‪:16-2‬‬
‫ﺑﺮآﯾﻨﺪ ﺳﯿﺴﺘﻢ ﻧﯿﺮو و ﮐﻮﭘﻞ وارده ﺑﺮ ﻣﮑﻌﺐ را ﺗﻌﯿﯿﻦ ﮐﻨﯿﺪ‪.‬‬

‫ﻣﺜﺎل ‪:17-2‬‬
‫ﺑﺮآﯾﻨﺪ ﻧﯿﺮوﻫﺎي ﻣﻮازي وارد ﺑﺮ ورق را ﻣﺤﺎﺳﺒﻪ ﮐﻨﯿﺪ‪ .‬ﻣﺴﺎﻟﻪ را ﺑﺎ رﻫﯿﺎﻓﺖ ﺑﺮداري ﺣﻞ ﮐﻨﯿﺪ‪.‬‬

‫ﻣﺜﺎل ‪:18-2‬‬
‫دو ﻧﯿﺮو و ﯾﮏ ﭘﯿﭽﻪ ﻣﻨﻔﯽ ﻧﺸﺎن داده ﺷﺪه را ﺑﺎ ﯾﮏ ﻧﯿﺮوي وارده ﺑﺮ ‪ A‬و ﮐﻮﭘﻞ ﻧﻈﯿﺮ ﺟﺎﯾﮕﺰﯾﻦ ﮐﻨﯿﺪ‪.‬‬

‫ﻣﺜﺎل ‪:19-2‬‬
‫ﭘﯿﭽﻪ ﯾﺎ رﻧﭻ )ﻧﯿﺮو و ﮔﺸﺘﺎور در راﺳﺘﺎي ﻫﻤﻮن ﻧﯿﺮو( ﻣﻌﺎدل ﺳﻪ ﻧﯿﺮوي وارد ﺑﺮ ورق را ﺑﯿﺎﺑﯿﺪ‪ .‬ﻣﺨﺘﺼﺎت ﻧﻘﻄـﻪ ‪ P‬از‬
‫ﺻﻔﺤﻪ ‪ x-y‬ﮐﻪ ﻧﯿﺮوي ﺑﺮآﯾﻨﺪ ﭘﯿﭽﻪ از آن ﻣﯽﮔﺬرد را ﺑﯿﺎﺑﯿﺪ‪.‬‬
c02.qxd 10/29/07 1:38 PM Page 91

Article 2/9 Resultants 91

Sample Problem 2/16 z

Determine the resultant of the force and couple system which acts on the 70 N.m
rectangular solid. 50 N
100 N.m

80 N
Solution. We choose point O as a convenient reference point for the initial 50 N
step of reducing the given forces to a force–couple system. The resultant force is 96
N.m
 R  ΣF  (80  80)i  (100  100)j  (50  50)k  0 N
O 1.2 m
The sum of the moments about O is 80 N

 MO  [50(1.6)  70]i  [80(1.2)  96] j  [100(1)  100]k x 1.6 m


1 m 100 N
 10i N  m

Hence, the resultant consists of a couple, which of course may be applied at any 100 N
point on the body or the body extended. y

Helpful Hints
 Since the force summation is zero, we conclude that the resultant, if it exists,
must be a couple.

 The moments associated with the force pairs are easily obtained by using the
M  Fd rule and assigning the unit-vector direction by inspection. In many
three-dimensional problems, this may be simpler than the M  r  F approach.

Sample Problem 2/17 50 N

Determine the resultant of the system of parallel forces which act on the 0.5 m
plate. Solve with a vector approach. z
x 0.5 m

O 0.35 m
Solution. Transfer of all forces to point O results in the force–couple system

R  ΣF  (200  500  300  50)j  350j N 500 N 0.35 m


MO  [50(0.35)  300(0.35)]i  [50(0.50)  200(0.50)]k y 200 N
 87.5i  125k N  m 300 N
The placement of R so that it alone represents the above force–couple system is
z
determined by the principle of moments in vector form
x
r  R  MO x
z
(xi  yj  zk)  350j  87.5i  125k r O
R
350xk  350zi  87.5i  125k
From the one vector equation we may obtain the two scalar equations R
MO
y
350x  125 and 350z  87.5

Hence, x  0.357 m and z  0.250 m are the coordinates through which the
Helpful Hint
line of action of R must pass. The value of y may, of course, be any value, as
 permitted by the principle of transmissibility. Thus, as expected, the variable y  You should also carry out a scalar
drops out of the above vector analysis. solution to this problem.
c02.qxd 10/29/07 1:38 PM Page 92

92 Chapter 2 Force Systems

Sample Problem 2/18 x


700 N
500 N
Replace the two forces and the negative wrench by a single force R applied
at A and the corresponding couple M. 25 N· m
60°
40°
z B
30 80 mm
Solution. The resultant force has the components mm
A 50 mm
100
mm 45°
[Rx  ΣFx] Rx  500 sin 40  700 sin 60  928 N 60 mm
600 N
[Ry  ΣFy] Ry  600  500 cos 40 cos 45  871 N 40 mm
120 mm
y
[Rz  ΣFz] Rz  700 cos 60  500 cos 40 sin 45  621 N

Thus, R  928i  871j  621k N


R
M
and R (928)2  (871)2  (621)2  1416 N Ans.

The couple to be added as a result of moving the 500-N force is


A
 [M  r  F] M500  (0.08i  0.12j  0.05k)  500(i sin 40
 j cos 40 cos 45  k cos 40 sin 45)
where r is the vector from A to B.

The term-by-term, or determinant, expansion gives Helpful Hints

M500  18.95i  5.59j  16.90k N  m  Suggestion: Check the cross-product


results by evaluating the moments
 The moment of the 600-N force about A is written by inspection of its x- and z- about A of the components of the
components, which gives 500-N force directly from the sketch.

M600  (600)(0.060)i  (600)(0.040)k  For the 600-N and 700-N forces it is


easier to obtain the components of
 36.0i  24.0k N  m their moments about the coordinate
directions through A by inspection
The moment of the 700-N force about A is easily obtained from the moments of of the figure than it is to set up the
the x- and z-components of the force. The result becomes cross-product relations.

M700  (700 cos 60)(0.030)i  [(700 sin 60)(0.060)  The 25-N  m couple vector of the
  (700 cos 60)(0.100)] j  (700 sin 60)(0.030)k wrench points in the direction oppo-
site to that of the 500-N force, and
 10.5i  71.4 j  18.19k N  m
we must resolve it into its x-, y-, and
z-components to be added to the
 Also, the couple of the given wrench may be written
other couple-vector components.

M  25.0(i sin 40  j cos 40 cos 45  k cos 40 sin 45)
 Although the resultant couple vec-
 16.07i 13.54 j  13.54k N  m tor M in the sketch of the resultants
is shown through A, we recognize
Therefore, the resultant couple on adding together the i-, j-, and k-terms of the that a couple vector is a free vector
four M’s is and therefore has no specified line
of action.
 M  49.4i  90.5j  24.6k N  m

and M  (49.4)2  (90.5)2  (24.6)2  106.0 N  m Ans.


c02.qxd 10/29/07 1:38 PM Page 93

Article 2/9 Resultants 93

Sample Problem 2/19 z


m
Determine the wrench resultant of the three forces acting on the bracket. 80 m
Calculate the coordinates of the point P in the x-y plane through which the resul- 60 mm
tant force of the wrench acts. Also find the magnitude of the couple M of the 40 N y
wrench.
100 mm
x
20 N
P
y
Solution. The direction cosines of the couple M of the wrench must be the
 same as those of the resultant force R, assuming that the wrench is positive. The 40 N
resultant force is x

R  20i  40j  40k N R  (20)2  (40)2  (40)2  60 N z


m
80 m
and its direction cosines are

y R
cos x  20/60  1/3 cos y  40/60  2/3 cos z  40/60  2/3 60 mm
x=
60
The moment of the wrench couple must equal the sum of the moments of mm
the given forces about point P through which R passes. The moments about P of
the three forces are P
100 mm
y = 40 mm
(M)Rx  20yk N  mm
M x
(M)Ry  40(60)i  40xk N  mm

(M)Rz  40(80  y)i  40(100  x)j N  mm Helpful Hint

and the total moment is


 We assume initially that the wrench
is positive. If M turns out to be neg-
ative, then the direction of the cou-
M  (800  40y)i  (4000  40x)j  (40x  20y)k N  mm
ple vector is opposite to that of the
The direction cosines of M are resultant force.

cos x  (800  40y)/M

cos y  (4000  40x)/M

cos z  (40x  20y)/M

where M is the magnitude of M. Equating the direction cosines of R and M gives

M
800  40y 
3

2M
4000 40x 
3

2M
40x 20y 
3

Solution of the three equations gives

M  2400 N  mm x  60 mm y  40 mm Ans.

We see that M turned out to be negative, which means that the couple vector is
pointing in the direction opposite to R, which makes the wrench negative.
‫ﻣﺜﺎلﻫﺎي ﻓﺼﻞ ﺳﻮم‪ :‬ﺗﻌﺎدل‬

‫ﻣﺜﺎل ‪ 1-3‬ﺗﺎ ‪ 4-3‬درﺑﺎره ﺗﻌﺎدل در ﺣﺎﻟﺖ دوﺑﻌﺪي‬

‫ﻣﺜﺎل ‪:1-3‬‬
‫اﻧﺪازه ﻧﯿﺮوي ‪ C‬و ‪ T‬ﮐﻪ ﻫﻤﺮاه ﺑﺎ ﺳﺎﯾﺮ ﻧﯿﺮوﻫﺎ ﺑﻪ اﺗﺼﺎل ﺧﺮﭘﺎي ﭘﻞ وارد ﻣﯽﺷﻮد را ﺗﻌﯿﯿﻦ ﮐﻨﯿﺪ‪.‬‬

‫ﻣﺜﺎل ‪:2-3‬‬
‫‪ 500‬را ﻣﻬﺎر ﮐﺮده‪ ،‬ﺑﯿﺎﺑﯿﺪ‪.‬‬ ‫ﮐﺸﺶ ﮐﺎﺑﻞ ‪ T‬ﮐﻪ ﺟﺮم‬
‫اﻧﺪازه ﻧﯿﺮوي ﮐﻞ وارد ﺑﺮ ﯾﺎﺗﺎﻗﺎن ﭘﻮﻟﯽ ‪ C‬را ﻣﺤﺎﺳﺒﻪ ﮐﻨﯿﺪ‪.‬‬

‫ﻣﺜﺎل ‪:3-3‬‬
‫‪ 100‬در اﺑﺘﺪا ﺑﻪ ﺻﻮرت اﻓﻘﯽ اﺳﺖ‪ .‬ﻣﻄﻠﻮﺑﺴﺖ ﺑﺎ اﺳﺘﻔﺎده از ﮐﺎﺑـﻞ در ‪ C‬اﻧﺘﻬـﺎي ‪ B‬را ﺑـﻪ‬ ‫ﺗﯿﺮ ‪ I‬ﺷﮑﻞ ﺑﺎ ﺟﺮم‬
‫ﺗﯿﺮ ﻧﺴﺒﺖ ﺑﻪ اﻓﻖ در وﺿـﻌﯿﺖ ﻧﻬـﺎﯾﯽ‪،‬‬ ‫‪ 3‬ﻧﺴﺒﺖ ﺑﻪ ‪ A‬ﺑﻠﻨﺪ ﺷﻮد‪ .‬ﮐﺸﺶ ﻻزم ‪ ،P‬واﮐﻨﺶ در ‪ A‬و زاوﯾﻪ‬ ‫اﻧﺪازه‬
‫را ﺑﯿﺎﺑﯿﺪ‪.‬‬

‫ﻣﺜﺎل ‪:4-3‬‬
‫اﻧﺪازه ﻧﯿﺮوي ﮐﺸﺶ در ﮐﺎﺑﻞ ﻧﮕﻬﺪارﻧﺪه و اﻧﺪازه ﻧﯿﺮو در ﭘـﯿﻦ )ﻣﻔﺼـﻞ( ‪ A‬از ﭼﺮﺛﻘﯿـﻞ ﺛﻘﻔـﯽ ﻧﺸـﺎن داده ﺷـﺪه را‬
‫‪ 95‬در ﻫﺮ ﻣﺘﺮ از ﻃﻮل ﺗﯿﺮ اﺳﺖ‪.‬‬ ‫ﺑﯿﺎﺑﯿﺪ‪ .‬ﺗﯿﺮ ‪ I‬ﺷﮑﻞ ﺑﺎ ﺟﺮم‬
c03.qxd 11/6/07 3:26 PM Page 127

Article 3/3 Equilibrium Conditions 127

Sample Problem 3/1 y

Determine the magnitudes of the forces C and T, which, along with the
other three forces shown, act on the bridge-truss joint. C

3 kN
x′
20°
Solution. The given sketch constitutes the free-body diagram of the isolated
 section of the joint in question and shows the five forces which are in equilibrium. y′
T

Solution I (scalar algebra). For the x-y axes as shown we have 40°
16 kN 8 kN
x
[ΣFx  0] 8  T cos 40  C sin 20  16  0
0.766T  0.342C  8 (a)
Helpful Hints
[ΣFy  0] T sin 40  C cos 20  3  0
 Since this is a problem of concur-
0.643T  0.940C  3 (b)
rent forces, no moment equation is
Simultaneous solution of Eqs. (a) and (b) produces necessary.

T  9.09 kN C  3.03 kN Ans.  The selection of reference axes to fa-


cilitate computation is always an im-
Solution II (scalar algebra). To avoid a simultaneous solution, we may use axes portant consideration. Alternatively
 x-y with the first summation in the y-direction to eliminate reference to T. Thus, in this example we could take a set
of axes along and normal to the di-
[ΣFy  0] C cos 20  3 cos 40  8 sin 40  16 sin 40  0 rection of C and employ a force sum-
mation normal to C to eliminate it.
C  3.03 kN Ans.
[ΣFx  0] T  8 cos 40  16 cos 40  3 sin 40  3.03 sin 20  0
T  9.09 kN Ans.

Solution III (vector algebra). With unit vectors i and j in the x- and y-direc-
tions, the zero summation of forces for equilibrium yields the vector equation

[ΣF  0] 8i  (T cos 40)i  (T sin 40)j  3j  (C sin 20)i


 (C cos 20)j  16i  0

Equating the coefficients of the i- and j-terms to zero gives

8  T cos 40  C sin 20  16  0


T sin 40  3  C cos 20  0

which are the same, of course, as Eqs. (a) and (b), which we solved above. P
20°
C
Solution IV (geometric). The polygon representing the zero vector sum of T 8 kN
the five forces is shown. Equations (a) and (b) are seen immediately to give the 3 kN
projections of the vectors onto the x- and y-directions. Similarly, projections onto 40°
the x- and y-directions give the alternative equations in Solution II. 16 kN
A graphical solution is easily obtained. The known vectors are laid off head-
 to-tail to some convenient scale, and the directions of T and C are then drawn to
close the polygon. The resulting intersection at point P completes the solution,  The known vectors may be added in
thus enabling us to measure the magnitudes of T and C directly from the draw- any order desired, but they must be
ing to whatever degree of accuracy we incorporate in the construction. added before the unknown vectors.
c03.qxd 11/6/07 3:26 PM Page 128

128 Chapter 3 Equilibrium

Sample Problem 3/2 T


θ = 30°
Calculate the tension T in the cable which supports the 500-kg mass with C
the pulley arrangement shown. Each pulley is free to rotate about its bearing,
and the weights of all parts are small compared with the load. Find the magni-
tude of the total force on the bearing of pulley C.
B

Solution. The free-body diagram of each pulley is drawn in its relative posi- A
tion to the others. We begin with pulley A, which includes the only known force.
With the unspecified pulley radius designated by r, the equilibrium of moments 500 kg
about its center O and the equilibrium of forces in the vertical direction require
 [ΣMO  0] T1r  T2r  0 T1  T2 T
30°
[ΣFy  0] T1  T2  500(9.81)  0 2T1  500(9.81) T1  T2  2450 N C
Fx
From the example of pulley A we may write the equilibrium of forces on pulley B T3
by inspection as Fy T4
T3  T4  T2/2  1226 N B y

For pulley C the angle   30 in no way affects the moment of T about the cen- T2
T1
ter of the pulley, so that moment equilibrium requires x
O
T  T3 or T  1226 N Ans. A

Equilibrium of the pulley in the x- and y-directions requires 500(9.81) N

[ΣFx  0] 1226 cos 30  Fx  0 Fx  1062 N Helpful Hint


[ΣFy  0] Fy  1226 sin 30  1226  0 Fy  613 N  Clearly the radius r does not influence
[F  Fx  Fy
2 2]
F (1062)2  (613)2  1226 N Ans. the results. Once we have analyzed a
simple pulley, the results should be
perfectly clear by inspection.

Sample Problem 3/3

The uniform 100-kg I-beam is supported initially by its end rollers on the
P
horizontal surface at A and B. By means of the cable at C it is desired to elevate
end B to a position 3 m above end A. Determine the required tension P, the reac-
tion at A, and the angle  made by the beam with the horizontal in the elevated
position. 6m C 2m
A B

Solution. In constructing the free-body diagram, we note that the reaction on P


B
the roller at A and the weight are vertical forces. Consequently, in the absence of 2m
other horizontal forces, P must also be vertical. From Sample Problem 3/2 we y 2m C
see immediately that the tension P in the cable equals the tension P applied to 3m
the beam at C. 4m
Moment equilibrium about A eliminates force R and gives A θ 100(9.81) N
x
 [ΣMA  0] P(6 cos )  981(4 cos )  0 P  654 N Ans.

Equilibrium of vertical forces requires R

[ΣFy  0] 654  R  981  0 R  327 N Ans. Helpful Hint


The angle  depends only on the specified geometry and is  Clearly the equilibrium of this paral-
sin   3/8   22.0 Ans. lel force system is independent of .
c03.qxd 11/6/07 3:26 PM Page 129

Article 3/3 Equilibrium Conditions 129

Sample Problem 3/4

Determine the magnitude T of the tension in the supporting cable and the
magnitude of the force on the pin at A for the jib crane shown. The beam AB is a
standard 0.5-m I-beam with a mass of 95 kg per meter of length.
0.25 m
A 25° B
0.5 m
Algebraic solution. The system is symmetrical about the vertical x-y plane
through the center of the beam, so the problem may be analyzed as the equilib- 1.5 m
0.12 m
rium of a coplanar force system. The free-body diagram of the beam is shown in
the figure with the pin reaction at A represented in terms of its two rectangular
10 kN
components. The weight of the beam is 95(103)(5)9.81  4.66 kN and acts
through its center. Note that there are three unknowns Ax, Ay, and T, which may 5m
be found from the three equations of equilibrium. We begin with a moment
equation about A, which eliminates two of the three unknowns from the equa- y T
 tion. In applying the moment equation about A, it is simpler to consider the mo- 25°
ments of the x- and y-components of T than it is to compute the perpendicular Ax
x
distance from T to A. Hence, with the counterclockwise sense as positive we
write Ay
4.66 kN
10 kN
 [ΣMA  0] (T cos 25)0.25  (T sin 25)(5  0.12)
Free-body diagram
 10(5  1.5  0.12)  4.66(2.5  0.12)  0
Helpful Hints
from which T  19.61 kN Ans.
 The justification for this step is
Equating the sums of forces in the x- and y-directions to zero gives
Varignon’s theorem, explained in
Art. 2/4. Be prepared to take full ad-
[ΣFx  0] Ax  19.61 cos 25  0 Ax  17.77 kN vantage of this principle frequently.
[ΣFy  0] Ay  19.61 sin 25  4.66  10  0 Ay  6.37 kN  The calculation of moments in two-
dimensional problems is generally
 [A  Ax2  Ay2] A  (17.77)2  (6.37)2  18.88 kN Ans. handled more simply by scalar alge-
bra than by the vector cross product
r  F. In three dimensions, as we will
see later, the reverse is often the case.
Graphical solution. The principle that three forces in equilibrium must be
concurrent is utilized for a graphical solution by combining the two known verti-  The direction of the force at A could
cal forces of 4.66 and 10 kN into a single 14.66-kN force, located as shown on the be easily calculated if desired. How-
modified free-body diagram of the beam in the lower figure. The position of this ever, in designing the pin A or in
resultant load may easily be determined graphically or algebraically. The inter- checking its strength, it is only the
section of the 14.66-kN force with the line of action of the unknown tension T magnitude of the force that matters.
defines the point of concurrency O through which the pin reaction A must pass. O
The unknown magnitudes of T and A may now be found by adding the forces T
head-to-tail to form the closed equilibrium polygon of forces, thus satisfying Ax 25°
their zero vector sum. After the known vertical load is laid off to a convenient
A
scale, as shown in the lower part of the figure, a line representing the given di- Ay
4.66 kN
rection of the tension T is drawn through the tip of the 14.66-kN vector. Like- 10 kN
wise a line representing the direction of the pin reaction A, determined from the
concurrency established with the free-body diagram, is drawn through the tail of 14.66 kN
the 14.66-kN vector. The intersection of the lines representing vectors T and A kN
establishes the magnitudes T and A necessary to make the vector sum of the 8 .88
A =1
forces equal to zero. These magnitudes are scaled from the diagram. The x- and 14.66 kN
y-components of A may be constructed on the force polygon if desired.
T=
19.6
1 kN

Graphical solution
‫ﻣﺜﺎل ‪ 5-3‬ﺗﺎ ‪ 7-3‬درﺑﺎره ﺗﻌﺎدل در ﺣﺎﻟﺖ ﺳﻪ ﺑﻌﺪي‬

‫ﻣﺜﺎل ‪:5-3‬‬
‫‪ 200‬ﺑﺎ ﯾﮏ ﺗﮑﯿﻪﮔﺎه ﮐﺎﺳﻪ‪-‬ﺳﺎﭼﻤﻪ ‪ A‬در ﺳﻄﺢ اﻓﻘﯽ ﻧﮕﻬﺪاري ﻣﯽﺷﻮد‪.‬‬ ‫ﻣﯿﻠﻪ ﯾﮑﻨﻮاﺧﺖ ﺑﺎ ﻃﻮل ‪ 7‬و ﺟﺮم‬
‫اﻧﺘﻬﺎي ‪ B‬ﺑﺎ دو دﯾﻮار ﻋﻤﻮدي ﺻﯿﻘﻠﯽ ﺗﮑﯿﻪ دارد‪ .‬ﻧﯿﺮوﻫﺎي وارده از ﮐﻒ و دﯾﻮارﻫﺎ را ﺑﺮ ﻣﯿﻠﻪ ﺑﯿﺎﺑﯿﺪ‪.‬‬

‫ﻣﺜﺎل ‪:6-3‬‬
‫ﻧﯿﺮوي ‪ 200‬در ﺟﻬﺖ ﻧﺸﺎن داده ﺷﺪه وارد ﻣﯽﺷﻮد‪ .‬ﯾﺎﺗﺎﻗﺎن ‪ A‬ﻣﯽﺗﻮاﻧﺪ ﻧﯿﺮو در ﺟﻬﺖ ﺷﻔﺖ ﻧﯿـﺰ ﺗﺤﻤـﻞ ﮐﻨـﺪ‬
‫وﻟﯽ ﯾﺎﺗﺎﻗﺎن ‪ B‬ﻓﻘﻂ ﻧﯿﺮو در ﺟﻬﺖ ﺷﻌﺎﻋﯽ‪ ،‬ﻧﯿﺮو در ﺟﻬﺖ ﻋﻤﻮد ﺑﺮ راﺳﺘﺎي ﻃﻮﻟﯽ ﺷﻔﺖ‪ ،‬ﺗﺤﻤﻞ ﻣﯽﮐﻨـﺪ‪ .‬ﺟـﺮم ‪m‬‬
‫ﻗﺎﺑﻞ ﻧﮕﻬﺪاري و ﻧﯿﺮوي ﺷﻌﺎﻋﯽ ﮐﻞ وارده ﺑﺮ ﺷﻔﺖ را ﺑﯿﺎﺑﯿـﺪ‪ .‬ﻓـﺮض ﮐﻨﯿـﺪ ﻫـﯿﭻ ﮐـﺪام از ﯾﺎﺗﺎﻗـﺎنﻫـﺎ ﻧﻤـﯽﺗﻮاﻧﻨـﺪ‬
‫ﮔﺸﺘﺎوري در راﺳﺘﺎي ﺧﻄﯽ ﻋﻤﻮد ﺑﺮ راﺳﺘﺎي ﻃﻮﻟﯽ ﺷﻔﺖ ﺗﺤﻤﻞ ﮐﻨﻨﺪ‪.‬‬

‫ﻣﺜﺎل ‪:7-3‬‬
‫ﻗﺎب ﻧﺸﺎن داده ﺷﺪه از ﺟﻮش ﭼﻨﺪ ﻣﯿﻠﻪ ﺳﺎﺧﺘﻪ و ﺑﺎ ﯾﮏ ﺗﮑﯿﻪﮔﺎه ﮐﺎﺳﻪ‪-‬ﺳﺎﭼﻤﻪ در ‪ A‬و ﺣﻠﻘﻪ ‪ B‬ﻣﻬﺎر ﺷﺪه اﺳـﺖ‪.‬‬
‫‪ 2‬و ﺑﺮاي ﺟﻠﻮﮔﯿﺮي از ﭼﺮﺧﺶ ﻗﺎب ﺣـﻮل راﺳـﺘﺎي‬ ‫)ﻣﯿﻠﻪ آزاداﻧﻪ در ﺣﻠﻘﻪ ﻣﯽﺗﻮاﻧﺪ دوران ﮐﻨﺪ( ﺗﺤﺖ ﻧﯿﺮوي‬
‫‪ AB‬از ﮐﺎﺑﻞ ﻣﻬﺎر اﺳﺘﻔﺎده ﺷﺪه اﺳﺖ‪ .‬ﺑﺎ ﻓﺮض ﭼﺸﻢﭘﻮﺷﯽ از وزن ﻗﺎب در ﺑﺮاﺑﺮ ﺑﺎر وارده‪ ،‬ﮐﺸﺶ در ﮐﺎﺑﻞ و ﻋﮑﺲ‪-‬‬
‫اﻟﻌﻤﻞ ﺗﮑﯿﻪﮔﺎﻫﯽ در ‪ A‬و ‪ B‬را ﺑﯿﺎﺑﯿﺪ‪.‬‬
c03.qxd 11/6/07 3:27 PM Page 150

150 Chapter 3 Equilibrium

Sample Problem 3/5


B
The uniform 7-m steel shaft has a mass of 200 kg and is supported by a ball-
and-socket joint at A in the horizontal floor. The ball end B rests against the
smooth vertical walls as shown. Compute the forces exerted by the walls and the 7m
floor on the ends of the shaft.

A
6m 2m
Solution. The free-body diagram of the shaft is first drawn where the contact
forces acting on the shaft at B are shown normal to the wall surfaces. In addition
to the weight W  mg  200(9.81)  1962 N, the force exerted by the floor on z
the ball joint at A is represented by its x-, y-, and z-components. These compo- Bx
By
 nents are shown in their correct physical sense, as should be evident from the re-
quirement that A be held in place. The vertical position of B is found from B
7  22  62  h2, h  3 m. Right-handed coordinate axes are assigned as shown.

3.
5
m
h
G
Vector solution. We will use A as a moment center to eliminate reference to
the forces at A. The position vectors needed to compute the moments about A are

3.
W = mg

5
y

m
rAG  1i  3j  1.5k m and rAB  2i  6j  3k m x
6m A
2m
where the mass center G is located halfway between A and B.
The vector moment equation gives Ay
Ax
Az
[ΣMA  0] rAB  (Bx  By)  rAG  W  0
(2i  6j  3k)  (Bxi  By j)  (i  3j  1.5k)  (1962k)  0

  
Helpful Hints
i j k i j k
2 6 3  1 3 1.5 0  We could, of course, assign all of the
Bx By 0 0 0 1962 unknown components of force in the
positive mathematical sense, in which
(3By  5890)i  (3Bx  1962)j  (2By  6Bx)k  0
case Ax and Ay would turn out to be
negative upon computation. The free-
Equating the coefficients of i, j, and k to zero and solving give
body diagram describes the physical
 Bx  654 N and By  1962 N Ans. situation, so it is generally preferable
The forces at A are easily determined by to show the forces in their correct
physical senses wherever possible.
[ΣF  0] (654  Ax)i  (1962  Ay)j  (1962  Az)k  0
 Note that the third equation 2By 
and Ax  654 N Ay  1962 N Az  1962 N
6Bx  0 merely checks the results of
Finally A  Ax2  Ay2  Az2 the first two equations. This result
could be anticipated from the fact
 (654)2  (1962)2  (1962)2  2850 N Ans.
that an equilibrium system of forces
concurrent with a line requires only
two moment equations (Category 2
Scalar solution. Evaluating the scalar moment equations about axes through under Categories of Equilibrium).
A parallel, respectively, to the x- and y-axes, gives
 We observe that a moment sum
[ΣMAx  0] 1962(3)  3By  0 By  1962 N about an axis through A parallel to
 [ΣMAy  0] 1962(1)  3Bx  0 Bx  654 N the z-axis merely gives us 6Bx 
2By  0, which serves only as a
The force equations give, simply, check as noted previously. Alterna-
[ΣFx  0] Ax  654  0 Ax  654 N tively we could have first obtained
Az from ΣFz  0 and then taken
[ΣFy  0] Ay  1962  0 Ay  1962 N our moment equations about axes
[ΣFz  0] Az  1962  0 Az  1962 N through B to obtain A x and A y.
c03.qxd 11/6/07 3:27 PM Page 151

Article 3/4 Equilibrium Conditions 151

Sample Problem 3/6 y


100
75 200 N
A 200-N force is applied to the handle of the hoist in the direction shown. 150
The bearing A supports the thrust (force in the direction of the shaft axis), while 100 250
B
60°
bearing B supports only radial load (load normal to the shaft axis). Determine A
the mass m which can be supported and the total radial force exerted on the 100 x 45°
shaft by each bearing. Assume neither bearing to be capable of supporting a mo-
ment about a line normal to the shaft axis. Radial
bearing z
Thrust
bearing
m
Dimensions in millimeters
Solution. The system is clearly three-dimensional with no lines or planes of
symmetry, and therefore the problem must be analyzed as a general space sys- x
tem of forces. A scalar solution is used here to illustrate this approach, although 70.7 N
a solution using vector notation would also be satisfactory. The free-body dia-
70.7 N
gram of the shaft, lever, and drum considered a single body could be shown by a
 space view if desired, but is represented here by its three orthogonal projections.
Az
The 200-N force is resolved into its three components, and each of the three z
views shows two of these components. The correct directions of Ax and Bx may be
seen by inspection by observing that the line of action of the resultant of the two Bx Ax y y 173.2 N
173.2 N
70.7-N forces passes between A and B. The correct sense of the forces Ay and By
cannot be determined until the magnitudes of the moments are obtained, so they Az 70.7 N
O x
z
are arbitrarily assigned. The x-y projection of the bearing forces is shown in 70.7 N
terms of the sums of the unknown x- and y-components. The addition of Az and By Ay Ay + By
the weight W  mg completes the free-body diagrams. It should be noted that Ax + Bx
the three views represent three two-dimensional problems related by the corre- mg = 9.81m mg = 9.81m
sponding components of the forces.

 From the x-y projection Helpful Hints

[ΣMO  0] 100(9.81m)  250(173.2)  0 m 44.1 kg Ans.  If the standard three views of ortho-
graphic projection are not entirely
From the x-z projection familiar, then review and practice
them. Visualize the three views as
[ΣMA 0] 150Bx  175(70.7)  250(70.7)  0 Bx  35.4 N the images of the body projected
onto the front, top, and end surfaces
of a clear plastic box placed over and
[ΣFx  0] Ax  35.4  70.7  0 Ax  35.4 N
aligned with the body.
 The y-z view gives
 We could have started with the x-z
[ΣMA  0] 150By  175(173.2)  250(44.1)(9.81)  0 By  520 N projection rather than with the x-y
projection.
[ΣFy  0] Ay  520  173.2  (44.1)(9.81)  0 Ay 86.8 N
 The y-z view could have followed im-
mediately after the x-y view since
[ΣFz  0] Az  70.7 N the determination of Ay and By may
The total radial forces on the bearings become be made after m is found.

[Ar  Ax2  Ay2] Ar  (35.4)2  (86.8)2  93.5 N Ans.  Without the assumption of zero mo-
ment supported by each bearing
 [B  Bx2  By2] B  (35.4)2  (520)2  521 N Ans. about a line normal to the shaft axis,
the problem would be statically in-
determinate.
c03.qxd 11/6/07 3:27 PM Page 152

152 Chapter 3 Equilibrium

Sample Problem 3/7 z


m C
3
The welded tubular frame is secured to the horizontal x-y plane by a ball- m
5
and-socket joint at A and receives support from the loose-fitting ring at B. Under 2. 4.5 B
the action of the 2-kN load, rotation about a line from A to B is prevented by the m
cable CD, and the frame is stable in the position shown. Neglect the weight of
the frame compared with the applied load and determine the tension T in the 2 kN
cable, the reaction at the ring, and the reaction components at A. 6m

2.5 m
A D
Solution. The system is clearly three-dimensional with no lines or planes of
symmetry, and therefore the problem must be analyzed as a general space sys- x 1m y
tem of forces. The free-body diagram is drawn, where the ring reaction is shown
in terms of its two components. All unknowns except T may be eliminated by a z
moment sum about the line AB. The direction of AB is specified by the unit
 vector n  2 1 2 (4.5j  6k)  15(3j  4k). The moment of T about AB E T Bx
6  4.5
is the component in the direction of AB of the vector moment about the point A F=
2 kN
and equals r1  T  n. Similarly the moment of the applied load F about AB is
r2  F  n. With CD  46.2 m, the vector expressions for T, F, r1, and r2 are Bz
r2 n

T
T (2i  2.5j  6k) F  2j kN Ay Ax
46.2
r1 D
 r1  i  2.5j m r2  2.5i  6k m Az y
x
The moment equation now becomes
z B
n
T
[ΣMAB  0] (i  2.5j)  (2i  2.5j  6k)  15(3j  4k)
46.2 r1 × T · n

 (2.5i  6k)  (2j)  15(3j  4k)  0 r1 × T

Completion of the vector operations gives


A
x y
48T
  20  0 T  2.83 kN Ans.
46.2
Helpful Hints
and the components of T become  The advantage of using vector nota-
tion in this problem is the freedom
Tx  0.833 kN Ty  1.042 kN Tz  2.50 kN to take moments directly about any
axis. In this problem this freedom
We may find the remaining unknowns by moment and force summations as permits the choice of an axis that
follows: eliminates five of the unknowns.

[ΣMz  0] 2(2.5)  4.5Bx  1.042(3)  0 Bx  0.417 kN Ans.  Recall that the vector r in the expres-
sion r  F for the moment of a force
[ΣMx  0] 4.5Bz  2(6)  1.042(6)  0 Bz  4.06 kN Ans. is a vector from the moment center to
any point on the line of action of the
[ΣFx  0] Ax  0.417  0.833  0 Ax  1.250 kN Ans. force. Instead of r1, an equally simple
l
 [ΣFy  0] Ay  2  1.042  0 Ay  3.04 kN Ans. choice would be the vector AC .

[ΣFz  0] Az  4.06  2.50  0 Az  1.556 kN Ans.


 The negative signs associated with
the A-components indicate that they
are in the opposite direction to those
shown on the free-body diagram.

You might also like